LSAT and Law School Admissions Forum

Get expert LSAT preparation and law school admissions advice from PowerScore Test Preparation.

 Administrator
PowerScore Staff
  • PowerScore Staff
  • Posts: 8917
  • Joined: Feb 02, 2011
|
#22941
Complete Question Explanation

Assumption-SN. The correct answer choice is (A)

The argument in this stimulus is basically that if a mental disorder results from a chemical imbalance, it can only be effectively treated with medication that will reduce or correct the chemical imbalance. The unstated presupposition here is that other forms of treatment (psychotherapy, say) cannot correct the chemical imbalance. In order to make the argument work, we need to find an answer choice that provides this missing assumption.

Answer choice (A): This is the correct answer choice. Assume the logical opposite of this statement: What if psychotherapy can produce effective reduction/correction of chemical imbalances that cause mental disorders? If this were true, then the author's argument fails — medication would not be the only way to treat chemically-caused disorders. So by the Assumption Negation test, this statement is necessary for the argument to work.

Answer choice (B): The argument makes no reference to the speed of results; it discusses only the effectiveness of treatment. So this answer choice is irrelevant.

Answer choice (C): We do not need to assume that most mental disorders have non-chemical causes. Even if most mental disorders do have chemical causes, the argument can still work — we would just be talking about a very large number of disorders.

Answer choice (D): Assume the logical opposite of this statement: What if psychotherapy were sometimes more effective than medication in treating patients with mental disorders? This deals generally with all mental disorders of various causes. This would not undermine the specific argument about medication being the only effective treatment for chemically-caused mental disorders. Thus, by the Assumption Negation test, this answer choice is not necessary for the argument.

Answer choice (E):"Effective treatment" was not defined in the stimulus. Does effective treatment imply merely a reduction of symptoms, or a deep correction of the underlying cause? This is not made clear. For this reason alone, we could reject this answer choice. Additionally, try the Assumption Negation test: Ask yourself, what if psychotherapy has an effect on mental disorders beyond just reduction of symptoms? Just like answer choice (D), this is a very general statement about all mental disorders, and it does not defeat the specific claim about medication being the only effective treatment for chemically-caused mental disorders.
 ericj_williams
  • Posts: 63
  • Joined: Jan 19, 2020
|
#84363
I think when you're talking about E your conflating something having an effect with it being effective. Nowhere is "effective" mentioned.
User avatar
 KelseyWoods
PowerScore Staff
  • PowerScore Staff
  • Posts: 1079
  • Joined: Jun 26, 2013
|
#84431
Hi ericj!

You're absolutely right that having an effect is not the same thing as being "effective." But that's the exact point that the explanation for answer choice (E) is trying to make!

Answer choice (E) tells us that psychotherapy has an effect--it reduces symptoms. But this is not necessary for the argument because the argument is about "effective treatment" and the stimulus never defines what "effective treatment" is. For all we know, reducing symptoms does count as "effective treatment." We can't be sure either way. And if you use the Assumption Negation Technique and negate answer choice (E) to say that psychotherapy does have other effects in addition to reducing statement, this does not attack the argument because even if it has those other effects, that still doesn't mean it qualifies as "effective treatment."

Hope this helps!

Best,
Kelsey

Get the most out of your LSAT Prep Plus subscription.

Analyze and track your performance with our Testing and Analytics Package.